Telechargé par loicanicetazegha

Exercices corrigés -Probabilités conditionnelles et indépendance

publicité
[email protected]
Rechercher sur le site...
Bibm@th
Rechercher sur le site...
Accueil Lycée Supérieur Bibliothèques Références Thèmes Forum
Bibliothèque d'exercicesBibliothèque de problèmesAutomatismes
Accueil
Lycée
Collège
Seconde
Supérieur
Math Sup 2020
Math Sup 2021
Math Spé
Capes
Agreg interne
BTS
Bibliothèques
Bibliothèque d'exercices
Bibliothèque de problèmes
Automatismes
Références
Dictionnaire
Biographie de mathématiciens
Formulaire
Lexique français/anglais
Thèmes
Cryptographie et codes secrets
Jeux et énigmes
Carrés magiques
Mathématiques au quotidien
Dossiers
Forum
https://www.bibmath.net/ressources/index.php?action=affiche&quoi=bde/proba/independance&type=fexo
01/12/2021, 10@34
Page 1 of 24
Télécharge ta lecture
favorite
La lecture virtuelle à portée de tous
Youscribe
Télécharger
Ressources mathématiques > Base de données d'exercices > Exercices de dénombrement - probabilités - statistiques >
Accéder à mon compte > Accéder à ma feuille d'exercices >
Exercices corrigés - Probabilités conditionnelles et indépendance
Indépendance
Exercice 1
- Indépendance et contexte [Signaler une erreur] [Ajouter à ma feuille d'exos]
Enoncé
1. Une urne contient 12 boules numérotées de 1 à 12. On en tire une hasard, et on considère les événements
A = "tirage d'un nombre pair'',
B = "tirage d'un multiple de 3''.
Les événements A et B sont-ils indépendants?
2. Reprendre la question avec une urne contenant 13 boules.
Indication
Écrire en termes d'ensembles les événements A,
B et A ∩ B. Puis calculer les probabilités.
Corrigé
1. On a :
A = { 2, 4, 6, 8, 10, 12}
B = { 3, 6, 9, 12}
A ∩ B = { 6, 12} .
On a donc P (A) = 1/ 2, P (B) = 1/ 3 et P (A ∩ B) = 1/ 6 = P (A)P (B). Les événements A et B sont indépendants.
2. Les événements A, B et A ∩ B s'écrivent encore exactement de la même façon. Mais cette fois, on a : P (A) = 6/ 13,
P (B) = 4/ 13 et P (A ∩ B) = 2/ 13 ≠ 24/ 169. Les événements A et B ne sont pas indépendants. C'est conforme à l'intuition.
Il n'y a plus la même répartition de boules paires et de boules impaires, et dans les multiples de 3 compris entre 1 et 13, la
répartition des nombres pairs et impairs est restée inchangée.
Exercice 2
- Indépendance deux à deux et indépendance mutuelle [Signaler une erreur] [Ajouter à ma feuille d'exos]
Enoncé
Votre voisine a deux enfants dont vous ignorez le sexe. On considère les trois événement suivants :
A="les deux enfants sont de sexes différents"
B="l'ainé est une fille"
C ="le cadet est un garçon".
Montrer que A, B et C sont deux à deux indépendants, mais ne sont pas mutuellement indépendants.
https://www.bibmath.net/ressources/index.php?action=affiche&quoi=bde/proba/independance&type=fexo
01/12/2021, 10@34
Page 2 of 24
Indication
Calculer la probabilité de
A, B, C , A ∩ B,...
Corrigé
1
P (B) = P (C) = 2 . Les quatre possibilités pour les deux enfants, supposées équiprobables, sont (F,G), (F,F), (G,G),
1
(G,F). On en déduit que P (A) = 2 .
1
Ensuite A ∩ B correspond à (F , G) et donc P (A ∩ B) = 4 = P (A)P (B). On en déduit que A et B sont indépendants. On prouve
de la même façon que A et C sont indépendants, et que B et C sont indépendants.
1
Enfin, A ∩ B ∩ C = B ∩ C et donc P (A ∩ B ∩ C) = 4 ≠ P (A)P (B)P (C). Les trois événements ne sont pas mutuellement
Clairement, on a
indépendants.
Exercice 3
- Probabilité d'une réunion et indépendance [Signaler une erreur] [Ajouter à ma feuille d'exos]
Enoncé
Soient A1 , … , An n événements d’un espace probabilisé (Ω, P ). On les suppose mutuellement indépendants et de probabilités
respectives pi = P (Ai ). Donner une expression simple de P (A1 ∪ ⋯ ∪ An ) en fonction de p1 , … , pn .
Application : on suppose qu'une personne est soumise à n expériences indépendantes les unes des autres et qu'à chaque expérience, elle
ait une probabilité p d'avoir un accident. Quelle est la probabilité qu'elle ait au moins un accident?
Indication
Passer par le complémentaire.
Corrigé
Si les événements
déduit que
¯¯¯¯¯¯
¯¯¯¯¯¯¯
A1 , … , An sont mutuellement indépendants, les événements complémentaires A1 , … , An le sont aussi. On en
¯¯¯¯¯¯
¯¯¯¯¯¯¯
P (A1 ∪ ⋯ ∪ An ) = 1 − P (A1 ∩ ⋯ ∩ An )
n
= 1 − ∏ P (Ai )
¯¯¯¯¯¯
i=1
n
= 1 − ∏(1 − pi ).
i=1
Dans le cas particulier proposé, la probabilité d'avoir au moins un accident vaut donc
Exercice 4
1 − (1 − p)n .
- Indépendance impossible [Signaler une erreur] [Ajouter à ma feuille d'exos]
Enoncé
On suppose qu'on a un espace probabilisé tel que l'univers Ω est un ensemble fini de cardinal un nombre premier p, et que le modèle
choisi soit celui de l'équiprobabilité. Prouver que deux événements A et B non triviaux (différent de ∅ et Ω) ne peuvent pas être
indépendants.
Indication
Raisonner par l'absurde, calculer le cardinal de
A ∩ B et utiliser le théorème de Gauss pour obtenir une contradiction.
Corrigé
Supposons qu'il existe A et B deux événements non triviaux indépendants. On note m le cardinal de A et n le cardinal de B. On a donc
P (A) = m/ p et P (B) = n/ p. Puisque A et B sont supposés indépendants, et toujours parce que le modèle adopté est celui de
l'équiprobabilité, on a :
card(A ∩ B) = p × P (A ∩ B) =
mn
.
p
Puisque le cardinal est un entier, p divise le produit mn, et par le théorème de Gauss, il divise l'un des deux, disons n. D'autre part,
puisque n ≤ p, ceci n'est possible que si n = 0 ou n = p. Autrement dit, seulement si B est ou l'événement certain, ou l'événement
impossible, c'est-à-dire un événement trivial.
Exercice 5
- Circuit électrique [Signaler une erreur] [Ajouter à ma feuille d'exos]
Enoncé
1. Soient A, B, C trois événements. Montrer que :
P (A ∪ B ∪ C) = P (A) + P (B) + P (C) − P (A ∩ B) − P (A ∩ C) − P (B ∩ C) + P (A ∩ B ∩ C).
2. On dispose de 3 composants électriques C1 , C2 et C3 dont la probabilité de fonctionnement est pi , et de fonctionnement
totalement indépendant les uns des autres. Donner la probabilité de fonctionnement du circuit
https://www.bibmath.net/ressources/index.php?action=affiche&quoi=bde/proba/independance&type=fexo
01/12/2021, 10@34
Page 3 of 24
2.1. si les composants sont disposés en série.
2.2. si les composants sont disposés en parallèle.
2.3. si le circuit est mixte : C1 est disposé en série avec le sous-circuit constitué de C2 et C3 en parallèle.
Indication
1. Appliquer trois fois la formule P (F ∪ G) =. . ..
2. On note Fi l'événement : ``le circuit Ci fonctionne''. Il faut calculer pour le premier cas
P (F1 ∪ F2 ∪ F3 ), et pour le troisième P (F1 ∩ (F2 ∪ F3 )).
P (F1 ∩ F2 ∩ F3 ), pour le second
Corrigé
1. On procède en deux temps. D'une part :
P ((A ∪ B) ∪ C) = P (A ∪ B) + P (C) − P ((A ∪ B) ∩ C).
Mais,
P ((A ∪ B) ∩ C) = P ((A ∩ C) ∪ (B ∩ C)) = P (A ∩ C) + P (B ∩ C) − P (A ∩ B ∩ C),
et
P (A ∪ B) = P (A) + P (B) − P (A ∩ B).
On appelle aussi ceci la formule du crible de Poincaré, elle se généralise avec plusieurs événements par récurrence.
2. On note Fi l'événement : ``le composant Ci fonctionne''. Par hypothèse, les événements Fi sont mutuellement indépendants.
Il faut calculer pour le premier cas P (F1 ∩ F2 ∩ F3 ) (le circuit formé par les trois composants disposés en série fonctionne si et
seulement si les trois composants fonctionnent), pour le second P (F1 ∪ F2 ∪ F3 ) (le circuit formé par les trois composants
disposés en parallèle fonctionne si et seulement si un des trois composants fonctionne), et pour le troisième P (F1 ∩ (F2 ∪ F3 )).
On a :
2.1. Par indépendance des événements :
P (F1 ∩ F2 ∩ F3 ) = p1 p2 p3 .
2.2. D'après la formule précédente, et par indépendance des événements :
P (F1 ∪ F2 ∪ F3 ) = p1 + p2 + p3 − p1 p2 − p1 p3 − p2 p3 + p1 p2 p3 .
2.3. L'événement
F2 ∪ F3 est indépendant de F1 . On a donc :
P (F1 ∩ (F2 ∪ F3 )) = P (F1 )P (F2 ∪ F3 ) = P (F1 )(P (F2 ) + P (F3 ) − P (F2 ∩ F3 ))
soit
P (F1 ∩ (F2 ∪ F3 )) = p1 (p2 + p3 − p2 p3 ) .
Exercice 6
-
[Signaler une erreur] [Ajouter à ma feuille d'exos]
Enoncé
Un livre contient 4 erreurs, numérotées de 1 à 4, et est relu par une suite de relecteurs pour correction. A chaque relecture, chaque
erreur est corrigée avec une probabilité 1/3. Les erreurs sont corrigées de manière indépendante les unes des autres, et les relectures
sont indépendantes les unes des autres.
1. Quelle est la probabilité que l’erreur numéro 1 ne soit pas corrigée à l’issue de la n-ième lecture ?
2. Quelle est la probabilité que le livre soit entièrement corrigé à l’issue de la n-ième lecture ? Combien faut-il de relectures pour
que cette probabilité soit supérieure à 0.9 ?
Indication
1.
2.
Corrigé
1. Notons Ai l'événement "l'erreur numéro 1 n'est pas corrigée par le i-ème relecteur". Alors on a P (Ai ) = 2/ 3 et les
événements Ai sont indépendants. On s'intéresse à la probabilité de l'événement A1 ∩ ⋯ ∩ An qui vaut donc
2
https://www.bibmath.net/ressources/index.php?action=affiche&quoi=bde/proba/independance&type=fexo
01/12/2021, 10@34
Page 4 of 24
n
P (A1 ∩ ⋯ ∩ An ) = ∏
i=1
2
2n
= n.
3
3
2. Notons
Bj l'événement "l'erreur numéro j n'est pas corrigée à l'issue de la n-ième relecture". D'après la question précédente,
4 ¯¯¯¯¯¯
n n
on a P (Bj ) = 2 / 3 pour j = 1, … , 4. Le livre est entièrement corrigé après la n-ième relecture si l'événement ⋂j=1 Bj est
réalisé. Les événements Bj étant indépendants, le livre est entièrement corrigé après n relectures avec une probabilité valant
4
∏ (1 −
j=1
Cette probabilité est supérieure à
4
2n
2n
n ) = (1 − n ) .
3
3
0, 9 si et seulement si
(1 −
4
2n
2 n
1/ 4
n ) ≥ 0.9 ⟺ ( ) ≤ 1 − (0.9)
3
3
⟺ n ln(2/ 3) ≤ ln(1 − 0.91/ 4 )
⟺ n≥
et donc ceci fonctionne dès que n
Exercice 7
ln(1 − 0.91/ 4 )
ln(2/ 3)
≥ 10.
- Indicatrice d'Euler [Signaler une erreur] [Ajouter à ma feuille d'exos]
Enoncé
Soit n > 1 un entier fixé. On choisit de manière équiprobable un entier x dans { 1, … , n} . Pour tout entier m ≤ n, on note Am
l'événement "m divise x". On note également B l'événement "x est premier avec n". Enfin, on note p1 , … , pr les diviseurs premiers de n.
1.
2.
3.
4.
5.
Exprimer B en fonction des Apk .
Pour tout entier naturel m qui divise n, calculer la probabilité de Am .
Montrer que les événements Ap1 , … , Apr sont mutuellement indépendants.
En déduire la probabilité de B.
Application : on note ϕ(n) le nombre d'entiers compris entre 1 et n qui sont premiers avec n. Démontrer que
r
ϕ(n) = n ∏ (1 −
k=1
1
).
pk
Indication
1. x est premier avec n si et seulement si aucun des diviseurs premiers de
2. Écrire quels sont tels les élements de Am .
3. Utiliser un résultat d'arithmétique.
4.
5.
n ne divise x.
Corrigé
1. On sait que
x est premier avec n si et seulement si aucun des diviseurs premiers de n ne divise x. On a donc :
¯¯¯¯¯¯¯¯
¯¯¯¯¯¯¯¯
B = Ap1 ∩ ⋯ ∩ Apr .
2. Puisque qu'on est en situation d'équiprobabilité, il suffit de calculer le cardinal de
m qui sont inférieurs ou égaux à n sont m, 2m, … , km. On a donc
P (Am ) =
3. Soit i1
Am . Mais si n = km, alors les multiples de
1
k
=
.
n
m
< ⋯ < im des entiers distincts choisis dans { 1, … , r} . On doit prouver que
P (Api1 ) … P (Apim ) = P (Api1 ∩ ⋯ ∩ Apim ).
Mais,
m
P (Api1 ) … P (Apim ) = ∏
j=1
1
.
p ij
,…,
https://www.bibmath.net/ressources/index.php?action=affiche&quoi=bde/proba/independance&type=fexo
…
01/12/2021, 10@34
Page 5 of 24
D'autre part, puisque pi1 , … , pim sont premiers entre eux deux à deux, un entier est multiple de pi1
multiple de chaque pij , j = 1, … , m. On en déduit que
… pim si et seulement s'il est
Api1 ∩ ⋯ ∩ Apim = Api1 …pim ,
soit
P (Api1 ∩ ⋯ ∩ Apim ) =
1
,
p i1 … p im
ce qui prouve le résultat voulu.
4. Les événements
¯¯¯¯¯¯¯¯
¯¯¯¯¯¯¯¯
Ap1 , … , Apr sont également indépendants. On en déduit que
r
r
P (B) = ∏ P (Apj ) = ∏ (1 −
¯¯¯¯¯¯¯¯
j=1
j=1
5. On sait aussi, par le modèle de l'équiprobabilité et puisque le cardinal de
P (B) =
1
).
pj
B est égal à ϕ(n), que
ϕ(n)
n
ce qui, grâce à la question précédente, donne le résultat voulu.
Exercice 8
- Deuxième lemme de Borel-Cantelli [Signaler une erreur] [Ajouter à ma feuille d'exos]
Enoncé
Soit (Ω, F , P) un espace probabilisé. Soit (An )n≥0 une suite d'événements indépendants. On note A = lim supn An . On suppose que
∑n P(An ) = +∞ et on souhaite prouver que P(A) = 1.
1. Préliminaire. Justifier que pour tout x > −1, ln(1 + x) ≤ x.
¯¯¯¯¯¯
¯¯¯¯¯¯
2. Soient n ≤ N . On note En,N = ⋂N
et En = ⋂k≥n Ak .
k=n Ak
2.1. Démontrer que (n étant fixé), limN→+∞ ln (P(En,N )) = −∞.
2.2. En déduire que P(En ) = 0.
2.3. En déduire que P(A) = 1.
Indication
1. Étudier
2.
2.1.
2.2.
2.3.
une fonction, ou utiliser un argument de convexité.
Utiliser l'inégalité précédente et l'hypothèse.
Composition des limites, et continuité monotone croissante.
Continuité monotone décroissante.
Corrigé
1. La fonction ln est concave. Sa courbe représentative est en-dessous de sa tangente au point d'abscisse 1. L'inégalité demandée
est juste la traduction analytique de cette propriété géométrique.
2.
2.1. Les événements
¯¯¯¯¯¯
Ak étant indépendants, il en est de même des événements Ak , et donc
N
N
P (En,N ) = ∏ P (Ak ) = ∏ (1 − P (Ak )).
¯¯¯¯¯¯
k=n
k=n
En utilisant l'inégalité précédente, on a
N
ln (P (En,N )) ≤ − ∑ P (Ak ).
k=n
Puisque ∑k≥n P (Ak )
= +∞, on en déduit le résultat.
(P (En,N )) tend vers 0 lorsque N tend vers l'infini (et n reste fixé).
2.2. Par composition par la fonction exponentielle,
Mais, la suite
(En,N )N est décroissante et
En = ⋂ En , N .
N≥n
https://www.bibmath.net/ressources/index.php?action=affiche&quoi=bde/proba/independance&type=fexo
01/12/2021, 10@34
Page 6 of 24
Ainsi,
P (En ) = lim P (En,N ) = 0.
N
2.3.
A s'écrit A =
¯¯¯¯¯¯¯
¯¯¯¯¯¯¯
⋂n En . La suite (En ) est décroissante et
¯¯¯¯¯¯¯
P (En ) = 1. Ainsi, on trouve que
¯¯¯¯¯¯¯
P (A) = lim P (En ) = 1.
n
Probabilités conditionnelles
Exercice 9
- Probabilités composées [Signaler une erreur] [Ajouter à ma feuille d'exos]
Enoncé
On considère une urne contenant 4 boules blanches et 3 boules noires. On tire une à une et sans remise 3 boules de l'urne. Quelle est la
probabilité pour que la première boule tirée soit blanche, la seconde blanche et la troisième noire?
Indication
Lire le titre de l'exercice.
Corrigé
On note Bi (resp. Ni ) l'événement : ``La i-ème boule tirée est blanche (resp. noire)''. On cherche à calculer
l'on va faire en utilisant la formule des probabilités composées :
P (B1 ∩ B2 ∩ N3 ), ce que
P (B1 ∩ B2 ∩ N3 ) = P (B1 )P (B2 |B1 )P (N3 |B1 ∩ B2 ).
P (B1 ) = 4/ 7, P (B2 |B1 ) = 3/ 6 (il reste 6 boules dont 3 blanches) et
6
P (N3 |B1 ∩ B2 ) = 3/ 5. Finalement, on obtient P (B1 ∩ B2 ∩ N3 ) = 35 .
Chacune des probabilités qui apparaît est facile à calculer, car
Exercice 10
- Deux ateliers [Signaler une erreur] [Ajouter à ma feuille d'exos]
Enoncé
Dans une entreprise deux ateliers fabriquent les mêmes pièces. L'atelier 1 fabrique en une journée deux fois plus de pièces que l'atelier
2. Le pourcentage de pièces défectueuses est 3% pour l'atelier 1 et 4% pour l'atelier 2. On prélève une pièce au hasard dans l'ensemble
de la production d'une journée. Déterminer
1. la probabilité que cette pièce provienne de l'atelier 1;
2. la probabilité que cette pièce provienne de l'atelier 1 et est défectueuse;
3. la probabilité que cette pièce provienne de l'atelier 1 sachant qu'elle est défectueuse.
Indication
Corrigé
Notons A l'événement ``la pièce provient de l'atelier 1'',
est défectueuse''.
B l'événement ``la pièce provient de l'atelier 2'' et D l'événement ``la pièce
1. L'énoncé nous dit que les 2/3 des pièces produites proviennent de l'atelier 1. Donc
2
1
P (A) = 2/ 3.
P (A ∩ D) = PA (D)P (A) = 0, 03 × 3 = 50 .
1
3. On démontre de même que P (B ∩ D) = 75 et donc que
2. On cherche
P (D) = P (A ∩ D) + P (B ∩ D) =
1
.
30
Ainsi, on a
PD (A) =
Exercice 11
P (A ∩ D)
P (D)
=
3
.
5
- A partir de dénombrement [Signaler une erreur] [Ajouter à ma feuille d'exos]
Enoncé
Une urne contient 8 boules blanches et 2 boules noires, indiscernables au toucher. On tire sans remise et successivement 3 boules de
cette urne.
1. Quelle est la probabilité qu'au moins une boule noire figure dans le tirage?
2. Sachant qu'au moins une boule noire figure dans le tirage, quelle est la probabilité que la première boule tirée soit noire?
https://www.bibmath.net/ressources/index.php?action=affiche&quoi=bde/proba/independance&type=fexo
01/12/2021, 10@34
Page 7 of 24
Indication
1. Dénombrer!
2. Dénombrer les tirages tels que la première boule est noire, et appliquer la définition des probabilités conditionnelles.
Corrigé
1. Distinguons les boules et ordonnons les tirages. Il y a 10 × 9 × 8 tirages possibles. Calculons maintenant le nombre de tirages
comprenant au moins une boule noire. Il y a deux possibilités :
Le tirage ne comporte qu'une seule boule noire. Il y a 3 façons de choisir la position de la boule noire (lors du premier tirage,
du deuxième, etc...), 2 choix pour cette boule, et ensuite 8 × 7 choix pour les deux boules blanches. En tout, il y a donc
3 × 2 × 8 × 7 tirages correspondants.
Le tirage comporte les deux boules noires. On choisit d'abord les deux tirages où on a pris les boules noires : on choisit 2
3
places parmi 3, soit (2) = 3. Ce choix fait, il y a 2 choix pour les boules noires, et 8 choix pour les boules blanches. Il y a
donc en tout 3 × 2 × 8 tels tirages.
Le nombre total de tirages est donc 6 × 8 × 8 et si on note
la probabilité de B est égale à
P (B) =
B l'événement "Au moins une boule noire figure dans le tirage", alors
6×8×8
8
=
.
10 × 9 × 8
15
Remarquons que, comme souvent dans ce type d'exercice, il est plus facile de déterminer la probabilité du complémentaire : en
effet, B̄ est l'événement "on ne tire que des boules blanches". Le nombre de tirages correspondant est 8 × 7 × 6 et donc
P (B̄) =
8×7×6
7×6
7
=
=
10 × 9 × 8
10 × 9
15
ce qui est bien la valeur attendue (ouf)!
2. Le dénombrement du nombre de tirages tels que la première boule soit noire est plus simple. Il y a exactement
tirages. Si A est l'événement : "La première boule tirée est noire", alors
P (A) =
2×9×8
2×9×8
1
= .
10 × 9 × 8
5
On cherche P (A|B). Par définition,
P (A|B) =
puisque
P (A ∩ B)
P (B)
=
P (A)
P (B)
=
3
8
A ⊂ B.
Exercice 12
- Probabilité conditionnelle égale à probabilité [Signaler une erreur] [Ajouter à ma feuille d'exos]
Enoncé
Soit (Ω, A, P) un espace probabilité et A un événement de probabilité non nulle. Donner une condition nécessaire et suffisante pour que
PA = P .
Indication
Que vaut PA (A)?
Corrigé
Supposons d'abord que PA = P. En particulier, P(A) = PA (A) = 1. Ainsi, A doit être un événement presque sûr.
Réciproquement, si on suppose que P(A) = 1, alors pour tout événement B, on a
PA (B) =
P(A ∩ B)
P(A)
= P(A ∩ B).
Maintenant, on sait que
P(A ∩ B) + P(Ā ∩ B) = P(B).
Mais puisque Ā ∩ B
⊂ Ā et que P(Ā) = 0, on en déduit que
P(A ∩ B) = P(B).
Ainsi,
PA = P si et seulement si A est un événement presque sûr.
Formule des probabilités totales
https://www.bibmath.net/ressources/index.php?action=affiche&quoi=bde/proba/independance&type=fexo
01/12/2021, 10@34
Page 8 of 24
Exercice 13
-
[Signaler une erreur] [Ajouter à ma feuille d'exos]
Enoncé
Un fumeur, après avoir lu une série de statistiques effrayantes sur les risques de cancer, décide d'arrêter de fumer; toujours d'après des
statistiques, on estime les probabilités suivantes : si cette personne n'a pas fumé le n-ième jour, alors la probabilité pour qu'elle ne fume
pas le jour suivant est 0,3; mais si elle a fumé le n-ième jour, alors la probabilité pour qu'elle ne fume pas le jour suivant est 0,9. Pour
n ≥ 0, on note Fn l'événement ``la personne fume le n-ième jour'' et pn la probabilité de Fn . En particulier on a p0 = 1.
1. Démontrer que pn+1 = −0,6pn + 0,7.
2. La personne va-t-elle s'arrêter de fumer?
Indication
1. Appliquer la formule des probabilités totales.
2. C'est une suite arithmético-géométrique.
Corrigé
1. D'après la formule des probabilités totales,
¯¯¯¯¯¯
P (Fn+1 ) = PFn (Fn+1 )P (Fn ) + P¯F¯¯¯¯¯(Fn+1 )P (Fn ).
n
L'énoncé nous dit que
¯¯¯¯¯¯¯¯¯¯¯
¯¯¯¯¯¯¯¯¯¯¯
PFn (Fn+1 ) = 0,9 et P¯F¯¯¯¯¯(Fn+1 ) = 0,3
n
d'ou
PFn (Fn+1 ) = 0,1 et P¯F¯¯¯¯¯(Fn ) = 0,7.
n
On a donc
pn+1 = 0,1pn + 0,7(1 − pn ) = −0,6pn + 0,7.
2. On a une suite arithmético-géométrique. L'équation ℓ = −0,6ℓ + 0,7 donne ℓ = 7/ 16. La suite (qn ) définie par qn = pn − ℓ
est une suite géométrique de raison 0,6, elle converge vers 0, et donc (pn ) tend vers 7/ 16. La probabilité que la personne fume
le n-ième jour tend vers 7/ 16. Elle ne va pas s'arrêter de fumer!
Exercice 14
- Déplacement d'un pion sur un triangle [Signaler une erreur] [Ajouter à ma feuille d'exos]
Enoncé
On considère trois points distincts du plan nommés A, B et C . Nous allons étudier le déplacement aléatoire d’un pion se déplaçant sur
ces trois points. A l’étape n = 0, on suppose que le pion se trouve sur le point A. Ensuite, le mouvement aléatoire du pion respecte les
deux règles suivantes :
le mouvement du pion de l’étape n à l’étape n + 1 ne dépend que de la position du pion à l’étape n;
pour passer de l'étape n à l'étape n + 1, on suppose que le pion a une chance sur deux de rester sur place, sinon il se déplace de
manière équiprobable vers l’un des deux autres points.
Pour tout n ∈ N, on note An l’évènement "le pion se trouve en A à l’étape n", Bn l’évènement "le pion se trouve en B à l’étape n" et Cn
l’évènement "le pion se trouve en C à l’étape n". On note également, pour tout n ∈ N,
⎛ an ⎞
an = P (An ), bn = P (Bn ), cn = P (Cn ) et Vn = ⎜ bn ⎟ .
⎝c ⎠
n
1.
2.
3.
4.
Calculer les nombres an , bn et cn pour n = 0, 1.
Pour n ∈ N, exprimer an+1 en fonction de an , bn et cn . Faire de même pour bn+1 et cn+1 .
Donner une matrice M telle que, pour tout n ∈ N, on a Vn+1 = MVn .
On admet que, pour tout n ∈ N, on a
4n + 2 4n − 1 4n − 1 ⎞
1 ⎛ n
n
n
M =
⎜4 − 1 4 + 2 4 − 1⎟.
3 ⋅ 4n ⎝ n
n
4 − 1 4 − 1 4n + 2 ⎠
n
En déduire une expression de an , bn et cn pour tout n ∈ N.
5. Déterminer les limites respectives des suites (an ), (bn ) et (cn ). Interpréter le résultat.
https://www.bibmath.net/ressources/index.php?action=affiche&quoi=bde/proba/independance&type=fexo
01/12/2021, 10@34
Page 9 of 24
Indication
1. Utiliser que b1 = c1 et a1 + b1 + c1 = 1.
2. Appliquer la formule des probabilités totales.
3.
4.
5.
Corrigé
1. Puisqu'en n = 0 le pion est en A, on a a0 = 1, b0 = 0 et c0 = 0. A l'étape n = 1, d'après les informations de l'énoncé,
a1 = 1/ 2, b1 = c1 . Puisque a1 + b1 + c1 = 1, on a b1 = c1 = 1/ 4.
2. Les événements An , Bn et Cn forment un système complet d'événements. D'après la formule des probabilités totales,
P (An+1 ) = PAn (An+1 )P (An ) + PBn (An+1 )P (Bn ) + PCn (An+1 )P (Cn ).
Comme à la question précédente, on a
PAn (An+1 ) = 1/ 2, PBn (An+1 ) = 1/ 4 et PCn (An+1 ) = 1/ 4. On en déduit que
an+1 =
1
1
1
a + b + c .
2 n 4 n 4 n
bn+1 =
1
1
1
a n + bn + c n
4
2
4
cn+1 =
1
1
1
a n + bn + c n .
4
4
2
En raisonnant de la même façon, ou par symétrie,
3. D'après la question précédente, la matrice
M=
convient.
4. On a V n
2 1 1⎞
1⎛
⎜1 2 1⎟
4⎝
1 1 2⎠
= M n V0 , ce qui donne
an
⎧
⎪
⎪
⎨ bn
⎪
⎪
⎩c
n
=
=
=
4n +1
3⋅4n
4n −2
3⋅4n
4n −2
.
3⋅4n
On remarque qu'on a bien an + bn + cn = 1.
5. Les suites (an ), (bn ) et (cn ) convergent toutes les trois vers 1/3. Au bout d'un très grand nombre de déplacements, les
probabilités que le pion sur chacun des 3 points sont presque égales.
Exercice 15
- Loi de Hardy-Weinberg [Signaler une erreur] [Ajouter à ma feuille d'exos]
Enoncé
Un gène présent dans une population est formé de 2 allèles A et a. Un individu peut donc avoir l’un des trois génotypes suivants : AA, Aa,
aa. Un enfant lors de la conception hérite d’un allèle de chacun de ses parents, chacun d’eux étant choisi au hasard. Ainsi si le père est
du type AA et la mère de type Aa, les enfants peuvent être du type AA ou Aa. On considère une population (génération 0) et on note p0 ,
q0 et r0 les proportions respectives de chacun des phénotypes AA, Aa et aa. On admet que les couples se forment au hasard
indépendamment des génotypes considérés.
1. Donner, en fonction de p0 , q0 et r0 la probabilité p1 qu'un enfant de la génération 1 ait un génotype AA.
2. Donner de même r1 , puis q1 .
3. Démontrer que p1 , q1 et r1 s'expriment uniquement en fonction de α = p0 − r0 . Que peut-on dire de p1 − r1 .
4. Donner les probabilités p2 , q2 et r2 qu'un enfant de la génération 2 ait pour génotype respectivement AA, Aa et aa. Que peut-on
conclure?
Indication
1. Utiliser la formule des probabilités totales.
Corrigé
https://www.bibmath.net/ressources/index.php?action=affiche&quoi=bde/proba/independance&type=fexo
01/12/2021, 10@34
Page 10 of 24
1. Notons A l'événement l'enfant a pour génotype AA, B1 l'événement "les deux parents ont pour génotype AA", B2 l'événement
"l'un des deux parents a pour génotype AA et l'autre a pour génotype Aa" et B3 l'événement "les deux parents ont pour génotype
Aa". On a
A = (A ∩ B1 ) ∪ (A ∩ B2 ) ∪ (A ∩ B3 )
et la réunion est une réunion disjointe. On en déduit que
P (A) = P (A ∩ B1 ) + P (A ∩ B2 ) + P (A ∩ B3 )
= P (B1 )P (A|B1 ) + P (B2 )P (A|B2 ) + P (B3 )P (A|B3 ).
P (B1 ) = p20 et P (A|B1 ) = 1. On a P (B2 ) = 2p0 q0 (attention! il faut
1
tenir compte ici qu'il y a deux cas, suivant que ce soit la mère ou le père qui ait le génotype AA) et P (A|B2 ) = 2 . On a enfin
1
2
P (B3 ) = q0 et P (A|B3 ) = 4 . On en déduit que
Calculons maintenant chacune des probabilités. On a
p1 = P (A) = p20 + p0 q0 +
2
1 2
1
q0 = (p0 + q0 ) .
4
2
2. Par symétrie, on a
r1 = (r0 +
Enfin, puisque p1
+ q1 + r1 = 1, on a
q1 = 1 − (p0 +
3. On sait que q0
2
1
q0 ) .
2
2
2
1
1
q0 ) − (r0 + q0 ) .
2
2
= 1 − p0 − r0 . En remplaçant dans l'expression de p1 ci-dessus, on trouve
p1 =
1
(1 + α)2 .
4
r1 =
1
(1 − α)2 .
4
De même, on trouve
Enfin,
q1 =
1 − α2
.
2
On remarque alors facilement que p1 − r1 = α = p0 − r0 .
4. Les calculs sont les mêmes de la génération 0 à la génération 1 ou de la génération 1 à la génération 2. Les formules donnant
p2 , q2 et r2 en fonction de p1 − r1 sont identiques aux formules donnant p1 , q1 et r1 en fonction de p0 − r0 . Puisque
p1 − r1 = p0 − r0 , on en déduit que p2 = p1 , q2 = q1 et r2 = r1 . Les suites (pn ), (qn ) et (rn ) sont donc stationnaires à partir
de la première génération. Dans cette situation (idéale), les proportions des trois génotypes ne varient donc pas dans le temps.
Exercice 16
- La rumeur [Signaler une erreur] [Ajouter à ma feuille d'exos]
Enoncé
Une information est transmise à l'intérieur d'une population. Avec une probabilité p, l'information reçue d'une personne est transmise
telle quelle à la personne suivante. Avec une probabilité 1 − p, l'information reçue d'une personne est transmise de façon contraire à la
personne suivante. On note pn la probabilité que l'information après n transmissions soit correcte.
1. Donner une relation de récurrence entre pn+1 et pn .
2. En déduire la valeur de pn en fonction de p et de n.
3. En déduire la valeur de limn pn . Qu'en pensez-vous?
Indication
1. Utiliser la formule des probabilités totales.
2. On a une suite arithmético-géométrique. Retirer la limite possible pour retrouver une suite géométrique.
Corrigé
https://www.bibmath.net/ressources/index.php?action=affiche&quoi=bde/proba/independance&type=fexo
01/12/2021, 10@34
Page 11 of 24
1. On note In l'événement : "l'information après
que
n transmissions est correcte''. D'après la formule des probabilités totales, on sait
¯¯¯¯¯
¯¯¯¯¯
P (In+1 ) = P (In+1 |In )P (In ) + P (In+1 |In )P (In ).
Mais, P (In+1 |In ) = p (l'information doit être transmise correctement) et
transmise). On en déduit que
¯¯¯¯¯
P (In+1 |In ) = 1 − p (l'information doit être mal
pn+1 = p × pn + (1 − p) × (1 − pn ) = (2p − 1)pn + (1 − p).
2. On a une suite arithmético-géométrique. Sa limite possible l vérifie
l = (2p − 1) × l + (1 − p) ⟺ l = 1/ 2.
On pose alors un
= pn −
1
et on vérifie que
2
un+1 = pn+1 −
On en déduit un
(un ) est géométrique de raison (2p − 1). En effet,
1
1
1
= (2p − 1)pn + (1 − p) − = (2p − 1) (pn − ) .
2
2
2
= (2p − 1)n u0 avec u0 = p0 − 1/ 2 = 1/ 2. On conclut que
pn =
1
1
+ (2p − 1)n .
2
2
3. On distingue alors trois cas :
Si p = 1, l'information est transmise presque sûrement correctement, et pn = 1 pour tout entier n.
Si p = 0, l'information est presque sûrement mal transmise, et p2n = 1, p2n+1 = 0 pour tout entier n.
Si p ∈]0, 1[, alors |2p − 1| < 1 et donc (pn ) converge vers 1/2. On n'a plus de traces de l'information initiale!
Exercice 17
- Sauts de puce [Signaler une erreur] [Ajouter à ma feuille d'exos]
Enoncé
Une particule se trouve à l'instant 0 au point d'abscisse a (a entier), sur un segment gradué de 0 à N (on suppose donc 0 ≤ a ≤ N ). A
chaque instant, elle fait un bond de +1 avec la probabilité p (0 < p < 1/ 2), ou un bond de −1 avec la probabilité q = 1 − p. Autrement
dit, si xn est l'abscisse de la particule à l'instant n, on a :
xn+1 = {
xn + 1 avec probabilité p
xn − 1 avec probabilité 1 − p.
Le processus se termine lorsque la particule atteint une des extrémités du segment (i.e. s'il existe xn avec xn = 0 ou xn = N ).
1. Écrire un algorithme qui simule cette marche aléatoire. En particulier, cet algorithme prendra en entrée l'abscisse a de départ,
la longueur N du segment, et produira en sortie un message indiquant si la marche s'arrête en 0 ou en N , et le nombre de pas
nécessaires pour que le processus s'arrête. On supposera qu'on dispose d'une fonction alea() qui retourne un nombre aléatoire
suivant une loi uniforme sur [0, 1].
2. On note ua la probabilité pour que la particule partant de a, le processus s'arrête en 0.
2.1. Que vaut u0 ? uN ?
2.2. Montrer que si 0 < a < N , alors ua = pua+1 + qua−1 .
2.3. En déduire l'expression exacte de ua .
3. On note va la probabilité pour que la particule partant de a, le processus s'arrête en N . Reprendre les questions précédentes
avec va au lieu de ua .
4. Calculer ua + va . Qu'en déduisez-vous?
Indication
1.
2.
2.1. Il suffit de décrypter l'énoncé.
2.2. Si la particule atteint le point a + 1, la probabilité que le processus s'arrête en 0 vaut
probabilités totales.
2.3. Réviser les suites récurrentes d'ordre 2.
3. Les mêmes indications que celles de la question précédente conviennent!
4. Que dire d'un événement de probabilité 1?
ua+1 . Appliquer la formule des
Corrigé
1. Voici un exemple d'algorithme.
https://www.bibmath.net/ressources/index.php?action=affiche&quoi=bde/proba/independance&type=fexo
01/12/2021, 10@34
Page 12 of 24
Demander N.
Demander a.
Si (a<0) ou (a>N) ou (N<0) alors afficher "Valeurs entrées incorrectes"
Sinon
0->pas
a->position
Tant que ( (position>0) et (position<N)) faire
pas+1->pas
Si (alea()<p)
alors position+1->position
sinon position-1->position
Fin Tant que.
Si (position==0)
alors Afficher "Fin du processus en 0"
sinon Afficher "Fin du processus en N"
Afficher "Nombre d'étapes :"
Afficher pas;
2.
2.1. Par définition, on a u0 = 1 (le processus commence en 0, il s'arrête immédiatement, en 0), et uN = 0 (le processus
commence en N , il s'arrête aussitôt, en N ).
2.2. On note A l'événement : "Partant de a, le processus s'arrête en 0", B l'événement : "Partant de a à l'instant 0, à
l'instant 1, la particule est en a + 1", et C l'événement : "Partant de a à l'instant 0, à l'instant 1, la particule est en a − 1",
de sorte que C = B̄. Par la formule des probabilités totales :
P (A) = P (B)P (A|B) + P (C)P (A|C).
Maintenant, puisqu'on part à l'instant t = 0 de a, on a P (B) = p et P (C) = q. D'autre part, si la particule est à l'instant 1
en a + 1, la probabilité que le processus s'arrête en 0 vaut ua+1 . On a donc : P (A|B) = ua+1 , et de même
P (A|C) = ua−1 . On en déduit la formule de récurrence :
ua = pua+1 + qua−1 .
2.3. Pour
a allant de 1 à N − 1, la suite (ua ) vérifie la formule de récurrence :
ua+1 =
1
q
ua − ua−1 .
p
p
L'équation caractéristique de cette récurrence est :
r2 −
1
q
r + = 0.
p
p
Cette équation du second degré admet deux solutions distinctes,
r1 = 1 et r2 =
q
p
(remarquer ici l'utilisation de l'hypothèse p ≠ 1/ 2 qui permet d'affirmer que les deux racines sont distinctes). Il existe donc
des réels λ et µ tels que, pour tout a dans { 0, … , N} , on ait :
q a
ua = λ + µ( ) .
p
Utilisant que u0
= 1 et uN = 0, on obtient :
ua =
q a
qN
pN
+
(
) .
q N − pN
pN − q N p
3. Le même raisonnement prouve que :
va = pva+1 + qva−1 .
La résolution de cette récurrence donne :
va =
4. On vérifie aisément que ua
q a
pN
pN
+
(
) .
pN − q N
q N − pN p
+ va = 1. Ceci signifie que, presque sûrement, le processus va s'arrêter.
https://www.bibmath.net/ressources/index.php?action=affiche&quoi=bde/proba/independance&type=fexo
01/12/2021, 10@34
Page 13 of 24
Exercice 18
- La puce [Signaler une erreur] [Ajouter à ma feuille d'exos]
Enoncé
Le sommets d'un triangle équilatéral sont numérotés 1, 2 et 3. Une puce saute de sommet en sommet de la façon suivante : si à l'instant
n elle se trouve sur un sommet donné, elle saute à l'instant n + 1 vers l'un des deux sommets voisins avec probabilité 1/2. On note Xn la
position de la puce à l'instant n.
1. On suppose dans cette question seulement que X0 suit une loi uniforme sur l'ensemble { 1, 2, 3} . Décrire la loi de X1 puis la loi
de Xn pour n ≥ 1.
⎛ P [Xn = 1] ⎞
On s'intéresse maintenant au cas général. On note Un = ⎜ P [Xn = 2] ⎟.
⎝ P [Xn = 3] ⎠
2. Montrer qu'il existe une matrice A que l'on déterminera telle que pour tout n ≥ 0, Un+1 = AUn .
3. Montrer qu'il existe une matrice orthogonale P , qu'on ne cherchera pas à calculer, telle que
0
0 ⎞
⎛1
0 ⎟Pt .
A = P ⎜ 0 −1/ 2
⎝0
0
−1/ 2 ⎠
4. En déduire que la suite (Un )n≥0 converge vers un vecteur U∞ vérifiant U∞ = AU∞ .
5. Que vaut U∞ ?
Indication
1.
2.
3.
4.
5.
Formule des probabilités totales.
Itou.
Réduction des matrices symétriques réelles.
Calculer An .
Résoudre l'équation précédente.
Corrigé
1. D'après la formule des probabilités totales, on a
P (X1 = 1) = P (X1 = 1|X0 = 1)P (X0 = 1) + P (X1 = 1|X0 = 2)P (X0 = 2) + P (X1 = 1|X0 = 3)P (X0 = 3)
1
1
1
1
1
=0+ × + × = .
2
2
3
3
3
1
P (X1 = 2) = P (X1 = 3) = 3 . La variable aléatoire X1 suit donc une loi uniforme sur
{ 1, 2, 3} . Par récurrence immédiate, la variable aléatoire Xn suivra aussi pour tout entier n la même loi.
De la même façon on trouve que
2. On applique encore la formule des probabilités totales. On a
P (Xn+1 = 1) = P (Xn+1 = 1|Xn = 1)P (Xn = 1) + P (Xn+1 = 1|Xn = 2)P (Xn = 2) + P (Xn+1 = 1|Xn = 3)P (Xn = 3)
1
1
= 0P (Xn = 1) + P (Xn = 2) + P (Xn = 3).
2
2
En effectuant le même calcul pour P (Xn+1
= 2) et P (Xn+1 = 3), on trouve que la matrice
⎛0
1
A=⎜
⎜
⎜2
⎝1
1
2
0
1
2
2
⎞
⎟
⎟
⎟
0⎠
1
2
1
2
convient.
3. La matrice A est symétrique réelle. Elle est donc diagonalisable, et si on note D la matrice de ses valeurs propres, il existe
une matrice orthogonale P telle que A = P DP t . Reste à démontrer que 1 est valeur propre de A, et que −1/ 2 est valeur
propre double. On peut calculer le polynôme caractéristique de A, ou remarquer que
valeur propre 1, et que
4. On a
Un =
An U
0
1
⎛1⎞
⎜ 1 ⎟ est vecteur propre de A pour la
⎝1⎠
A + 2 I3 est de rang 1, ce qui assure que −1/ 2 est valeur propre d'ordre 2.
= P Dn P t U0 où on a conservé les notations de la question précédente. Mais
⎛1 0 0⎞
D → B = ⎜0 0 0⎟.
⎝0 0 0⎠
n
(
)
=
=
https://www.bibmath.net/ressources/index.php?action=affiche&quoi=bde/proba/independance&type=fexo
=
.
01/12/2021, 10@34
Page 14 of 24
(Un ) converge vers P BP t U0 = U∞ . De la relation Un+1 = AUn , on tire en passant à la limite que U∞ = AU∞ .
⎛1⎞
5. On sait que 1 est valeur propre de A d'ordre 1, et on a déjà trouvé un vecteur propre précédemment, à savoir ⎜ 1 ⎟. U∞ est
⎝1⎠
donc proportionnel à ce vecteur propre. Mais, puisque la somme des lignes de chaque vecteur Un fait 1, il en est de même de la
Donc
somme des lignes des vecteurs de
Exercice 19
U∞ . Ainsi, on a U∞
⎛
=⎜
⎜
⎜
⎝
1
3
1
3
1
3
⎞
⎟
⎟
⎟.
⎠
- Marche aléatoire sur un triangle [Signaler une erreur] [Ajouter à ma feuille d'exos]
Enoncé
1. Question préliminaire : Soit M la matrice
⎛
M=⎜
⎜
⎜
⎝
1
3
1
3
1
3
1
12
7
12
1
3
0
1
0
⎞
⎟
.
⎟
⎟
⎠
Démontrer que M est diagonalisable, et trouver P inversible et D diagonale telles que M = P DP −1 .
2. On considère une particule se déplaçant à chaque seconde sur l'un des trois sommets A, B et C d'un triangle suivant le procédé
suivant :
si la particule se trouve en B, elle y reste;
si la particule se trouve en A, elle se rend la seconde suivante sur l'un des trois sommets de façon équiprobable;
si la particule se trouve en C , à la seconde suivante, elle y reste une fois sur trois, sinon elle va en B sept fois plus souvent
qu'en A.
A la première seconde, la particule se pose de façon équiprobable sur un des trois sommets. Pour tout n ≥ 1, on note An (resp. Bn
, Cn ) l'événement "à la n-ième seconde, la particule se trouve en A" (resp. B et C ), et on note an , bn et cn les probabilités
respectives de An , Bn et Cn .
Que valent a1 , b1 et c1 ?
3. Donner une relation de récurrence entre an+1 , bn+1 , cn+1 et an , bn et cn .
⎛ an ⎞
4. On note, pour n ≥ 1, Xn le vecteur Xn = ⎜ bn ⎟. Vérifier que Xn+1 = MXn .
⎝c ⎠
n
5. En déduire la valeur de an , bn et cn .
6. Étudier la convergence des suites (an ), (bn ) et (cn ).
Indication
1. Calculer le polynôme caractéristique....
2.
3. Utiliser la formule des probabilités totales.
4.
5. Utiliser la diagonalisation de M .
6.
Corrigé
1. Le calcul du polynôme caractéristique de la matrice
colonne. On trouve que le polynôme caractéristique est
χM (x) =
Les valeurs propres sont
1
M est grandement facilité en développant par rapport à la deuxième
1
3
(1 − x) ( − 6x + 9x2 ) .
9
4
1
1, 2 , 6 . Une matrice P possible est
⎛ 0 −1 1 ⎞
P = ⎜1 3
1 ⎟
⎝ 0 −2 −2 ⎠
d'inverse
⎛
1
1
1
⎞
https://www.bibmath.net/ressources/index.php?action=affiche&quoi=bde/proba/independance&type=fexo
01/12/2021, 10@34
Page 15 of 24
P −1
1
⎛ 1
−
=⎜
⎜ 2
⎝ 1
2
⎞
1
0 −4 ⎟.
⎟
1
0 − ⎠
1
1
4
1
2. Par hypothèse d'équiprobabilité, on a a1 = b1 = c1 = 3 .
3. On va utiliser la formule des probabilités totales. L'énoncé nous donne
PAn (An+1 ) = PAn (Bn+1 ) = PAn (Cn+1 ) =
1
3
PBn (An+1 ) = PBn (Cn+1 ) = 0, PBn (Bn+1 ) = 1
PCn (An+1 ) =
1
1
PC (Bn+1 ), PCn (Cn+1 ) = .
7 n
3
Utilisant que
PCn (An+1 ) + PCn (Bn+1 ) + PCn (Cn+1 ) = 1,
on trouve finalement que
PCn (An+1 ) =
Les trois événements
maintenant
1
7
, PCn (Bn+1 ) =
.
12
12
An , Bn et Cn constituant un système complet d'événements, la formule des probabilités totale donne
an+1 = P (An+1 ) = PAn (An+1 )P (An ) + PBn (An+1 )P (Bn ) + PCn (An+1 )P (Cn )
1
1
c .
= an +
12 n
3
De même, on trouve que
bn+1 =
1
1
1
7
a n + bn +
c , c
= an + c n .
12 n n+1
3
3
3
4. C'est évident!
5. On a Xn = P Dn P −1 X0 et comme on sait calculer
an =
Dn , on trouve après calculs
1
1 n
1 n
1
1 n
1 n
(( ) + ( ) ) , bn = 1 − (3( ) + ( ) )
2
2
2
2
6
6
1 n
1 n
cn = ( ) − ( ) .
2
6
6. Les suites
(an ) et (cn ) tendent vers 0, la suite (bn ) tend vers 1.
Formule de Bayes
Exercice 20
- Clés USB [Signaler une erreur] [Ajouter à ma feuille d'exos]
Enoncé
Le gérant d'un magasin d'informatique a reçu un lot de clés USB. 5% des boites sont abîmées. Le gérant estime que :
60% des boites abîmées contiennent au moins une clé défectueuse.
98% des boites non abîmées ne contiennent aucune clé défectueuse.
Un client achète une boite du lot. On désigne par A l'événement : ``la boite est abîmée'' et par D l'événement ``la boite achetée
contient au moins une clé défectueuse''.
1. Donner les probabilités de P (A), P (Ā), P (D|A), P (D|Ā), P (D̄|A) et P (D̄|Ā). En déduire la probabilité de D.
2. Le client constate qu'un des clés achetées est défectueuse. Quelle est la probabilité pour qu'il ait acheté une boite abîmée?
Indication
1. Traduire les informations de l'énoncé, utiliser la formule des probabilités totales.
2. Utiliser la formule de Bayes.
https://www.bibmath.net/ressources/index.php?action=affiche&quoi=bde/proba/independance&type=fexo
01/12/2021, 10@34
Page 16 of 24
Corrigé
1. L'énoncé donne directement
P (A) = 0, 05, d'où P (Ā) = 0, 95, P (D|A) = 0, 6 et P (D̄|Ā) = 0, 98. On en déduit :
P (D̄|A) = 1 − P (D|A) = 0, 4
P (D|Ā) = 1 − P (D̄|Ā) = 0, 02.
D'après la formule des probabilités totales :
P (D) = P (A)P (D|A) + P (Ā)P (D|Ā) =
2. On obtient
P (A|D) grâce à la formule de Bayes :
P (A|D) =
Exercice 21
49
.
1000
P (A ∩ D)
P (D)
=
P (D|A)P (A)
P (D)
=
30
.
49
- QCM [Signaler une erreur] [Ajouter à ma feuille d'exos]
Enoncé
Un questionnaire à choix multiples propose m réponses pour chaque question. Soit p la probabilité qu'un étudiant connaisse la bonne
réponse à une question donnée. S'il ignore la réponse, il choisit au hasard l'une des réponses proposées. Quelle est pour le correcteur la
probabilité qu'un étudiant connaisse vraiment la bonne réponse lorsqu'il l'a donnée?
Indication
Utiliser la formule de Bayes.
Corrigé
On note :
B = {L'étudiant donne la bonne réponse}
C = {L'étudiant connait la bonne réponse} .
On cherche PB (C)
= P (C|B), et l'énoncé donne :
P (C) = p, P (B|C) = 1, P (B|C̄) =
1
.
m
D'après la formule des probabilités totales, on a :
P (B) = P (C)P (B|C) + P (C̄)P (B|C̄) =
(m − 1)p + 1
.
m
D'après la formule de Bayes :
P (C|B) =
Exercice 22
P (B|C)P (C)
P (B)
=
mp
.
1 + (m − 1)p
- Dés pipés [Signaler une erreur] [Ajouter à ma feuille d'exos]
Enoncé
On dispose de 100 dés dont 25 sont pipés. Pour chaque dé pipé, la probabilité d'obtenir le chiffre 6 lors d'un lancer vaut 1/ 2.
1. On tire un dé au hasard parmi les 100 dés. On lance ce dé et on obtient 6. Quelle est la probabilité que ce dé soit pipé?
2. Soit n ∈ N∗ . On tire un dé au hasard parmi 100 dés. On lance ce dé n fois et on obtient n fois le chiffre 6. Quelle est la
probabilité pn pour que ce dé soit pipé. Interpréter le résultat.
Indication
1. Formule de Bayes!
2.
Corrigé
1. Notons
T l'événement : ``le dé est pipé'' et C l'événement ``le lancer amène un 6''. On cherche PC (T ). On va calculer cette
( , ¯)
https://www.bibmath.net/ressources/index.php?action=affiche&quoi=bde/proba/independance&type=fexo
01/12/2021, 10@34
Page 17 of 24
(T , T¯) est un système complet d'événements de probabilités non nulles,
¯
avec P (T ) = 25/ 100 = 1/ 4 et P (T ) = 1 − P (T ) = 3/ 4. L'énoncé nous dit aussi que PT (C) = 1/ 2 et bien sûr
PT¯ (C) = 1/ 6. La formule de Bayes donne alors
probabilité en utilisant la formule de Bayes. En effet,
PC (T ) =
=
=
P (T )PT (C)
PT (C)P (T ) + PT¯ (C)P (T¯)
1
2
×
1
.
2
1
4
×
1
2
1
4
+
1
6
×
3
4
n
2. Introduisons les événements Ck définis par ``le k-ième lancer amène un 6'' et D = ⋂k=1 Ck . Nous cherchons PD (T ) que l'on
calcule toujours par la formule de Bayes. Il faut juste remarquer que maintenant, par indépendance des événements Ck ,
PT (D) = (1/ 2)n et PT¯ (D) = (1/ 6)n . On trouve donc :
PD (T ) =
=
=
En particulier, (pn ) tend vers 1 lorsque
certainement pipé.
Exercice 23
P (T )PT (D)
PT (D)P (T ) + PT¯ (D)P (T¯)
×(2)
1
4
1
n
(2) ×
1
1 + 3−n+1
n
+(6) ×
1
4
1
n
1
3
4
.
n tend vers l'infini, ce qui est conforme à l'intuition : plus n est grand, plus le dé est très
- Pièces défectueuses [Signaler une erreur] [Ajouter à ma feuille d'exos]
Enoncé
Une usine fabrique des pièces, avec une proportion de 0,05 de pièces défectueuses. Le contrôle des fabrications est tel que :
si la pièce est bonne, elle est acceptée avec la probabilité 0,96.
si la pièce est mauvaise, elle est refusée avec la probabilité 0,98.
On choisit une pièce au hasard et on la contrôle.Quelle est la probabilité
1. qu'il y ait une erreur de contrôle?
2. qu'une pièce acceptée soit mauvaise?
Indication
1. Décomposer l'événement en "bonne et refusée" et "mauvaise et acceptée", ces deux événements étant incompatibles.
2. C'est la formule de Bayes!
Corrigé
1. On note
A l'événement "la pièce est acceptée par le contrôle", et B l'événement "la pièce est bonne". L'événement E "Il y a
¯¯¯¯
¯¯¯¯
A ∩ B et A ∩ B. Ces deux derniers événements sont incompatibles, on a donc :
une erreur au contrôle" se décompose en
¯¯¯¯
¯¯¯¯
P (E) = P (A ∩ B) + P (A ∩ B).
¯¯¯¯
Maintenant, P (A ∩ B)
¯¯¯¯
¯¯¯¯
¯¯¯¯
¯¯¯¯
¯¯¯¯ ¯¯¯¯
= P (A|B)P (B). Or, P (B) = 0, 05, et P (A|B) = 1 − P (A|B) = 0, 02. De même, on a
¯¯¯¯
¯¯¯¯
¯¯¯¯
P (A ∩ B) = P (A|B)P (B) et on a P (A|B) = 1 − P (A|B) = 0, 04. On obtient finalement :
P (E) = 0, 95 × 0, 04 + 0, 05 × 0, 02.
2. Dans cette question, on cherche
utiliser la formule de Bayes.
¯¯¯¯
P (B|A) alors que l'on connait les probabilités conditionnelles sachant B. Ceci nous invite à
¯¯¯¯
¯¯¯¯
https://www.bibmath.net/ressources/index.php?action=affiche&quoi=bde/proba/independance&type=fexo
01/12/2021, 10@34
Page 18 of 24
¯¯¯¯
¯¯¯¯
P (B)P (A|B)
¯¯¯¯
P (B|A) =
¯¯¯¯
¯¯¯¯
P (B)P (A|B) + P (B)P (A|B)
0, 05 × 0, 02
1
=
≃ 0, 001.
=
0, 95 × 0, 96 + 0, 05 × 0, 02
913
Exercice 24
- Compagnie d'assurance [Signaler une erreur] [Ajouter à ma feuille d'exos]
Enoncé
Une compagnie d'assurance répartit ses clients en trois classes R1 , R2 et R3 : les bons risques, les risques moyens, et les mauvais risques.
Les effectifs de ces trois classes représentent 20% de la population totale pour la classe R1 , 50% pour la classe R2 , et 30% pour la classe
R3 . Les statistiques indiquent que les probabilités d'avoir un accident au cours de l'année pour une personne de l'une de ces trois classes
sont respectivement de 0.05, 0.15 et 0.30.
1. Quelle est la probabilité qu'une personne choisie au hasard dans la population ait un accident dans l'année?
2. Si M.Martin n'a pas eu d'accident cette année, quelle est la probabilité qu'il soit un bon risque?
Indication
1. Appliquer la formule des probabilités totales.
2. Appliquer la formule de Bayes.
Corrigé
1. On note A l'événement "avoir un accident dans l'année". Comme les trois classes
population. On peut appliquer la formule des probabilités totales :
R1 , R2 et R3 réalisent une partition de la
P (A) = P (A|R1 )P (R1 ) + P (A|R2 )P (R2 ) + P (A|R3 )P (R3 )
= 0, 05 × 0, 2 + 0, 15 × 0, 5 + 0, 3 × 0, 3
= 0, 175.
2. On cherche la probabilité d'être dans
de Bayes donne :
¯¯¯¯
R1 sachant qu'on n'a pas eu d'accident, c'est-à-dire la probabilité P (R1 |A). La formule
¯¯¯¯
¯¯¯¯
P (R1 |A) =
La probabilité
finalement :
¯¯¯¯
¯¯¯¯
P (A)
.
¯¯¯¯
¯¯¯¯
P (A) se calcule par la formule P (A) = 1 − P (A), tandis que l'énoncé donne P (A|R1 ) = 0, 95. On obtient
¯¯¯¯
P (R1 |A) =
Exercice 25
P (A|R1 )P (R1 )
0, 95 × 0, 2
= 0, 23.
1 − P (A)
- La forêt [Signaler une erreur] [Ajouter à ma feuille d'exos]
Enoncé
Une forêt se compose de trois types d'arbres : 30% sont des chênes, 50% des peupliers, et 20% des hêtres. Suite à une tempête, une
maladie se déclare et touche 10% des chênes, 4% des peupliers, et 25% des hêtres. Sachant qu'un arbre est malade, quelle est la
probabilité que ce soit un chêne? un peuplier? un hêtre?
Indication
Utiliser la formule de Bayes.
Corrigé
Notons M l'événement "l'arbre étudié est malade", et respectivement C, H, Q l'événement "l'arbre étudié est un chêne (resp. un hêtre,
un peuplier)". On cherche à connaitre PM (C). Par la formule de Bayes,
PM (C) =
PC (M)P (C)
PC (M)P (C) + PH (M)P (H) + PQ (M)P (Q)
0, 1 × 0, 3
=
0, 1 × 0, 3 + 0, 04 × 0, 5 + 0, 25 × 0, 2
3
.
=
10
https://www.bibmath.net/ressources/index.php?action=affiche&quoi=bde/proba/independance&type=fexo
01/12/2021, 10@34
Page 19 of 24
De la même façon, on trouve que
PM (H) =
5
2
, PM (Q) =
.
10
10
On vérifie bien que la somme des probabilités vaut 1.
Exercice 26
- Tests de dépistage [Signaler une erreur] [Ajouter à ma feuille d'exos]
Enoncé
Vous êtes directeur de cabinet du ministre de la santé. Une maladie est présente dans la population, dans la proportion d'une personne
malade sur 10000. Un responsable d'un grand laboratoire pharmaceutique vient vous vanter son nouveau test de dépistage : si une
personne est malade, le test est positif à 99%. Si une personne n'est pas malade, le test est positif à 0, 1%. Autorisez-vous la
commercialisation de ce test?
Indication
Calculer la probabilité qu'une personne est malade si elle a une réponse positive au test.
Corrigé
Les chiffres donnés ont l'air excellent, mais ils donnent l'inverse de ce que l'on souhaite. Le problème est plutôt le suivant : si une
personne a une réponse positive au test, est-elle malade? C'est la formule de Bayes qui permet de remonter le chemin. Précisément, on
−4
note M l'événement "La personne est malade'', et T l'événement "le test est positif''. Les données dont on dispose sont P (M) = 10 ,
P (T |M) = 0, 99 et P (T |M̄) = 0, 001. On cherche P (M|T ). La formule de Bayes donne :
P (M|T ) =
=
P (T |M)P (M)
P (T |M)P (M) + P (T |M̄)P (M̄)
10−4 × 0, 99
10−4 × 0, 99 + 10−3 × 0, 9999
≃ 0, 09.
C'est catastrophique! La probabilité pour qu'une personne positive au test soit effectivement malade est inférieure à 10%. Le test
engendre donc beaucoup de faux-positifs (personnes positives au test, mais non malades). C'est tout le problème des maladies assez rares
: les test de dépistage doivent être extrêmement fiables. Remarquons par ailleurs ici une bonne illustration du vieil adage des
statisticiens : on peut faire dire n'importe quoi aux chiffres, cf le laboratoire pharmaceutique!
Exercice 27
- Menteur! [Signaler une erreur] [Ajouter à ma feuille d'exos]
Enoncé
Vous jouez à pile ou face avec un autre joueur. Il parie sur pile, lance la pièce, et obtient pile. Quelle est la probabilité pour qu'il soit un
tricheur?
Indication
Introduire la proportion x de tricheurs dans la population, et donner, à l'aide du théorème de Bayes, une estimation de la probabilité qu'il
soit un tricheur en fonction de x. Il faudra convenir de certaines probabilités de bon sens.
Corrigé
Soit x la proportion de tricheurs dans la population. On note respectivement P , F , H, T les événements "le joueur obtient pile'', "le
joueur obtient face'', "Le joueur est honnête'', "le joueur est un tricheur''. Il semble raisonnable de convenir que P (P |H) = 1/ 2 et
P (F |H) = 1/ 2 et P (P |T ) = 1 (un tricheur fait vraiment ce qu'il veut!). On cherche donc P (T |P ). De la formule de Bayes, on déduit
:
P (T |P ) =
P (P |T )P (T )
P (P |T )P (T ) + P (P |H)P (H)
Le résultat est plus ou moins réconfortant suivant la proportion de tricheurs
Exercice 28
=
2x
x
=
.
x+1
x + 1/ 2(1 − x)
x dans la population!
- Machines à sous [Signaler une erreur] [Ajouter à ma feuille d'exos]
Enoncé
Un joueur décide de jouer aux machines à sous. Il va jouer sur deux machines A et B qui sont réglées de la facon suivante :
la probabilité de gagner sur la machine A est de
la probabilité de gagner sur la machine B est de
1
5
1
10
;
.
Comme le joueur soupçonne les machines d'avoir des réglages différents, mais ne sait pas laquelle est la plus favorable, il décide
d'adopter la stratégie suivante :
il commence par choisir une machine au hasard ;
https://www.bibmath.net/ressources/index.php?action=affiche&quoi=bde/proba/independance&type=fexo
01/12/2021, 10@34
Page 20 of 24
après chaque partie, il change de machine s'il vient de perdre, il rejoue sur la même machine s'il vient de gagner.
On définit pour tout k ≥ 1 les événements suivants :
Gk : "Le joueur gagne la k-ième partie".
Ak : "La k-ième partie se déroule sur la machine A".
1.
2.
3.
4.
5.
Écrire un algorithme qui simule le déroulement de n parties et retourne la proportion de parties gagnées parmi ces n parties.
Determiner la probabilité de gagner la première partie.
Déterminer la probabilité de gagner la deuxième partie.
Sachant que la deuxième partie a été gagnée, quelle est la probabilité que la première partie ait eu lieu sur la machine A?
Soit k ≥ 1.
5.1. Exprimer P (Gk ) en fonction de P (Ak ).
7
9
5.2. Montrer que P (Ak+1 ) = − 10 P (Ak ) + 10 .
5.3. En déduire P (Ak ) puis P (Gk ) en fonction de k.
n
5.4. Pour n ≥ 1, on pose Sn = ∑k=1 P (Gk ). Calculer Sn puis déterminer la limite de
Sn
n
quand n → +∞.
Indication
1.
2. Appliquer la formule des probabilités totales.
3. Faire un arbre.
4. Appliquer la formule de Bayes.
5.
5.1. Appliquer la formule des probabilités totales.
5.2. Il y a deux éventualités pour jouer sur la machine A la k + 1-ième partie suivant ce qui se passe à la k-ième.
5.3. Reconnaitre une suite arithmético-géométrique un+1 = aun + b. On résout ℓ = aℓ + b puis on étudie vn = un
5.4. Somme d'une suite géométrique.
− ℓ.
Corrigé
1. Voici un algorithme possible. La variable machine représente la machine avec laquelle on joue (0 pour la machine A, 1 pour la
machine B).
VARIABLES
n EST_DU_TYPE NOMBRE
machine EST_DU_TYPE NOMBRE
total EST_DU_TYPE NOMBRE
k EST_DU_TYPE NOMBRE
DEBUT_ALGORITHME
LIRE n
SI (random()<0.5) ALORS
DEBUT_SI
machine PREND_LA_VALEUR 0
FIN_SI
SINON
DEBUT_SINON
machine PREND_LA_VALEUR 1
FIN_SINON
POUR k ALLANT_DE 1 A n
DEBUT_POUR
SI (machine==0) ALORS
DEBUT_SI
SI (random()<=0.2) ALORS
DEBUT_SI
total PREND_LA_VALEUR total+1
FIN_SI
SINON
DEBUT_SINON
machine PREND_LA_VALEUR 1
FIN_SINON
FIN_SI
SINON
DEBUT_SINON
SI (random()<=0.1) ALORS
DEBUT_SI
total PREND_LA_VALEUR total+1
FIN_SI
SINON
DEBUT_SINON
machine PREND_LA_VALEUR 0
FIN_SINON
FIN_SINON
https://www.bibmath.net/ressources/index.php?action=affiche&quoi=bde/proba/independance&type=fexo
01/12/2021, 10@34
Page 21 of 24
FIN_POUR
AFFICHERCALCUL total/n
FIN_ALGORITHME
2. D'après la formule des probabilités totales, on a
¯¯¯¯¯¯
1
1
1
1
3
× +
× =
.
2
2
5
10
20
¯¯¯¯¯¯
P (G1 ) = P (G1 |A1 )P (A1 ) + P (G1 |A1 )P (A1 ) =
3. Le plus facile, pour calculer G2 , est de réaliser un arbre de probabilités avec 8 feuilles. La première bifurcation correspond au
choix initial de la machine. La deuxième bifurcation au résultat possible du premier tirage. La troisième bifurcation au résultat
possible du troisième tirage. On trouve finalement :
P (G2 ) =
1 1
1
1
1 1
1
9
1
31
4
( × + ×
)+ (
×
+
× )=
.
2 5
2
5
5
10
10
10
10
5
200
4. Appliquons la formule de Bayes : on a
P (A1 |G2 ) =
La probabilité conditionnelle
P (A1 ∩ G2 )
P (G2 )
=
P (G2 |A1 )P (A1 )
P (G2 )
.
P (G2 |A1 ) se calcule également avec un arbre, et on trouve
P (G2 |A1 ) =
1
3
4
+
=
.
25
50
25
P (A1 |G2 ) =
3
200
12
×
=
.
50
31
31
On en déduit que
5.
5.1. On applique la formule des probabilités totales comme à la première question. On trouve
¯¯¯¯¯¯
¯¯¯¯¯¯
P (Gk ) = P (Gk |Ak )P (Ak ) + P (Gk |Ak )P (Ak )
1
1
= P (Ak ) +
(1 − P (Ak ))
5
10
1
(1 + P (Ak )).
=
10
5.2. On joue sur la machine A la k + 1-ième partie si et seulement si
on joue sur la machine A la k-ième partie et on gagne;
on joue sur la machine B la k-ième partie et on perd.
On a donc
¯¯¯¯¯¯¯
¯¯¯¯¯¯
P (Ak+1 ) = P (Gk ∩ Ak ) + P (Gk ∩ Ak )
¯¯¯¯¯¯¯ ¯¯¯¯¯¯
¯¯¯¯¯¯
= P (Gk |Ak )P (Ak ) + P (Gk |Ak )P (Ak )
1
9
(1 − P (Ak ))
= P (Ak ) +
5
10
9
−7
P (Ak ) +
.
=
10
10
5.3. On reconnait une suite arithmético-géométrique. L'équation aux limites possibles
ℓ=
donne
ℓ=
9
. On introduit ensuite
17
vk = P (Ak ) −
9
−7ℓ
+
10
10
9
et on vérifie facilement que
17
vk+1 =
On a donc vk
= ( 10 )
−7
k−1
−7
vk .
10
v1 et donc
−1
https://www.bibmath.net/ressources/index.php?action=affiche&quoi=bde/proba/independance&type=fexo
01/12/2021, 10@34
Page 22 of 24
P (Ak ) = (−
=
7
)
10
k−1
(P (A1 ) −
9
9
)+
17
17
k−1
−1
9
7
(− )
+
.
10
34
17
On en déduit alors que
P (Gk ) =
k−1
13
1
7
−
(− ) .
85
340
10
5.4. On reconnait la somme d'une suite géométrique. On a donc
13
1 ⎛ 1 − (− 10 )
⎜
n−
∑ P (Gk ) =
85
340 ⎜ 1 + 7
k=1
⎝
10
7
n
=
n
⎞
⎟
⎟
⎠
n
13n
1
7
−
(1 − (− ) ) .
85
34 × 17
10
13
En particulier, on a que Sn / n tend vers 85 , ce qui est proche du résultat trouvé en faisant tourner l'algorithme de la
première question pour une grande valeur de n.
Variables aléatoires indépendantes
Exercice 29
- Probabilité que la somme soit grande [Signaler une erreur] [Ajouter à ma feuille d'exos]
Enoncé
Soit (Xi )i∈N une suite de variables aléatoires indépendantes suivant toutes la même loi. On pose, pour n ≥ 1, Sn = X1 + ⋯ + Xn et on
considère a ∈ R. Démontrer l'équivalence suivante :
P (X1 ≥ a) > 0 ⟺ ∀n ≥ 1, P (Sn ≥ na) > 0.
Indication
Relier l'événement
Sn ≥ na aux événements X1 ≥ a, X2 ≥ a, … , Xn ≥ a.
Corrigé
Le sens réciproque est trivial puisqu'il correspond uniquement au cas
n = 1. Pour le sens direct, on remarque que
(Sn ≥ na) ⊃ (X1 ≥ a) ∩ (X2 ≥ a) ∩ ⋯ ∩ (Xn ≥ a)
ce qui implique que
P (Sn ≥ na) ≥ P ((X1 ≥ a) ∩ (X2 ≥ a) ∩ ⋯ ∩ (Xn ≥ a)).
Les variables aléatoires étant indépendantes, on en déduit que
n
P (Sn ≥ na) ≥ ∏ P (Xj ≥ a).
j=1
Enfin, puisque les Xi ont toutes la même loi que
X1 , on a
n
P (Sn ≥ na) ≥ (P (X1 ≥ a)) .
Ainsi, si
P (X1 ≥ a) > 0, on a bien P (Sn ≥ na) > 0 pour tout entier n ≥ 1, ce qui prouve le sens direct.
Discussions des forums
Taux d'évolution, pourcentage
Syracuse! encore
Exercice sur les séries
Base de topologie
crible en python
crible Python et C++
Suite de suites d'ensembl …
https://www.bibmath.net/ressources/index.php?action=affiche&quoi=bde/proba/independance&type=fexo
01/12/2021, 10@34
Page 23 of 24
calcul d'un périmètre .
Une grande erreur en math …
Changement de variables d …
Exercice de série issu de …
Isomorphisité d'un groupe
limite d'une suite
Problème de Cauchy
Action de groupes et déno …
Accéder aux forums
Mathématicienne du mois
Yvonne Choquet-Bruhat (1923 - )
Toutes les biographies
Signaler une erreur/Nous contacterMentions LégalesConfidentialité
ContactConfidentialitéMentions légales
https://www.bibmath.net/ressources/index.php?action=affiche&quoi=bde/proba/independance&type=fexo
01/12/2021, 10@34
Page 24 of 24
Téléchargement